Esto sustituye a mi respuesta original, que era defectuosa pero llevaba en la dirección correcta (creo).
Dejaré que la serie comience en algún $n_0\ge2$ La razón de ello se verá más adelante.
Ampliar en $\cos(n)/\ln(n)$ (con $\lvert\cos(n)/\ln(n)\rvert < 1$ para $n \ge 2$ ):
\[\sum_ {n=n_0}^ \infty \sum_ {k=0}^ \infty \frac {(-1)^n}{ \ln (n)} \frac { \cos ^k(n)}{ \ln ^k(n)}\]
Intercambiemos los sumandos y preocupémonos después de si estuvo bien:
\[\sum_ {k=0}^ \infty \sum_ {n=n_0}^ \infty \frac {(-1)^n}{ \ln (n)} \frac { \cos ^k(n)}{ \ln ^k(n)}\]
Ahora la suma interna converge según Prueba de Dirichlet . De hecho podemos proceder como en la demostración de la prueba de Dirichlet para acotar su límite (suprimiendo los índices $k$ mientras trabajamos en fijo $k$ ):
\[a_n= \ln ^{-k-1}(n)\] \[b_n=(-1)^n \cos ^k(n)= \sum_ {j=0}^k \alpha_ {jk} \cos ((j + \pi )n)\;,\]
con $\lvert\alpha_{jk}\rvert \le 1$ utilizando la fórmula de reducción de potencia para el coseno y absorbiendo el factor $(-1)^n$ en los cosenos sumando $\pi$ a su frecuencia.
\[B_m= \sum_ {n=n_0}^m \; b_n\]
\[\lvert B_m \rvert = \left\lvert\sum_ {n=n_0}^m \sum_ {j=0}^k \alpha_ {jk} \cos ((j + \pi )n) \right\rvert \le \sum_ {j=0}^k \left\lvert\sum_ {n=n_0}^m \cos ((j + \pi )n) \right\rvert \le \sum_ {j=0}^k \frac {2}{1 - \cos (j + \pi )}=:M_k\]
\[\left\lvert\sum_ {n=n_0}^m a_n b_n \right\rvert = \left\lvert B_m a_{m+1}+ \sum_ {n=n_0}^{m}B_n(a_n-a_{n+1}) \right\rvert\le\ ] \[ \le M_k a_{m+1} + \sum_ {n=n_0}^{m}M_k(a_n-a_{n+1})= M_k a_{m+1} + M_k(a_{n_0}-a_{m+1}) = M_ka_{n_0}\]
Así, podemos aplicar la prueba comparativa con las siguientes series:
\[\sum_ {k=0}^ \infty M_k \ln ^{-k-1}(n_0)\]
Esto demuestra por qué necesitamos empezar en algún $n_0$ en lugar de $2$ lo que no funcionaría ya que $\ln 2 < 1$ .
Ahora todo depende del comportamiento de $M_k$ . El coseno se acerca arbitrariamente a $1$ ; cuando $j$ entra en $\epsilon$ de un múltiplo impar de $\pi$ entonces $1/(1-\cos (j + \pi)) \approx 1/(1-(1 - \epsilon^2/2))= 2/\epsilon^2$ . Heurísticamente hablando, podríamos esperar que esto ocurriera cada $1/\epsilon$ enteros, por lo que en promedio estos picos no destruirían la decadencia a largo plazo con $\ln^{-k-1}(n_0)$ (que puede hacerse cuantitativamente, aunque no cualitativamente, más fuerte aumentando $n_0$ ). Sin embargo, no sé cómo hacer que este argumento sea riguroso, o si eso es siquiera posible dada la "imprevisibilidad" de $\pi$ . Es interesante que esta prueba pueda depender de los detalles de $\pi$ de esta manera. [Actualización: Gracias a George Lowther por los comentarios que apuntan a esta página de MathWorld lo que da un límite superior a la medida de irracionalidad de $\pi$ (nunca lo había oído antes :-). Que esto sea finito implica que $M_k$ puede estar limitada por una potencia de $k$ y, por tanto, la suma exterior converge (uniformemente), creo que eso completa la demostración].
Suponiendo que se pueda resolver el problema de los picos, aún tenemos que volver a la intercambiabilidad de las sumas. Este teorema muestra que podemos intercambiar las sumas si la convergencia de
\[\sum_ {k=0}^ \infty \sum_ {n=n_0}^m \frac {(-1)^n}{ \ln (n)} \frac { \cos ^k(n)}{ \ln ^k(n)}\]
es uniforme en $m$ -- que lo es, ya que $m$ se eliminó en la derivación del límite $M_ka_{n_0}$ .
En resumen, esta derivación muestra que se puede demostrar que la serie converge si los picos irregulares en $1/(1-\cos (j + \pi))$ que se producen porque $j$ se acerca arbitrariamente a impar múltiplos de $\pi$ puede demostrarse que no destruye la convergencia de la suma exterior sobre $k$ .
Gracias por esta interesante pregunta :-).
8 votos
Esto se mencionó como ejemplo de una serie para la que se necesita saber bastante sobre aproximaciones racionales a $\pi$ para demostrar la convergencia, en un comentario a la siguiente pregunta MO. mathoverflow.net/questions/54758/
0 votos
@George, gracias por tu comentario. Me encantaría conocer los detalles si alguien aquí los conoce....
0 votos
No he visto una prueba de esto, pero yo piense en el resultado se obtendrá a partir de la equidistribución, mostrando que los términos positivos y negativos de la serie se anulan en gran medida entre sí. Probablemente tendrás que utilizar el hecho de que $\pi$ tiene una medida de irracionalidad finita para obtener un límite lo suficientemente bueno como para demostrar la convergencia, como con la respuesta de David Speyer a esta pregunta anterior: math.stackexchange.com/questions/2270/
1 votos
Si $f$ es una función suave con periodo 1 y $\alpha$ tiene medida de irracionalidad finita, entonces se puede acotar $\sum_{n=1}^Nf(\alpha n)-N\int_0^1f(x)dx$ . A continuación, puede vincular $\sum_{n=1}^N(-1)^nf(\alpha n)$ . Creo que esto podría funcionar para esta suma aplicando equidistribución al $\cos n$ parte, pero aún no he revisado los detalles.
1 votos
Existe otra serie cuya convergencia depende de sutiles detalles de $\pi$ es decir, cuán raramente un seno o un coseno es "extremo" cuando se evalúa en valores enteros, y que es el Serie Flint Hills $$\sum_{n=1}^\infty \frac{1}{n^3(\sin n)^2}$$ llamado así por el topónimo por C. A. Pickover .